You are on page 1of 9

Version 001 Electromagnetism tubman (12126) This print-out should have 21 questions.

ns. Multiple-choice questions may continue on the next column or page nd all choices before answering. Conceptual 16 Q15 001 10.0 points The magnetic eld at the equator points north. If you throw a positively charged object (for example, a baseball with some electrons removed) to the east, what is the direction of the magnetic force on the object? 1. Toward the west 2. Downward Correct answer: 1.2648 1011 N. 3. Upward correct 4. Toward the east Explanation: Use the right-hand rule: point your index nger east and your middle nger north. Your thumb points upward (representing the force on a positively charged object). Hewitt CP9 24 E27 002 10.0 points Can an electron at rest in a magnetic eld be set into motion by the magnetic eld? What if it were at rest in an electric eld? 1. None of these 2. yes; no 3. no for both 4. yes for both 5. no; yes correct 6. It depends on the intensity of the elds, which is not provided in the problem. Explanation: An electron has to move across lines of magnetic eld in order to feel a magnetic AP B 1998 MC 21 004 10.0 points f = qvB = (1.60218 1019 C) (1.73119 108 m/s)(0.456 T) = 1.2648 1011 N . Let : V = 1.73119 108 m/s B = 0.456 T . and Explanation:

force, so an electron at rest in a stationary magnetic eld will feel no force to set it in motion. However, an electron in an electric eld will accelerate regardless of its current state of motion. Electron in a Magnetic Field 02 003 10.0 points An electron in a vacuum is rst accelerated by a voltage of 85200 V and then enters a region in which there is a uniform magnetic eld of 0.456 T at right angles to the direction of the electrons motion. What is the force on the electron due to the magnetic eld?

The kinetic energy gained after acceleration 1 is KE = me v 2 = qe V , so the velocity is 2 v= = 2qe V m 2(1.60218 1019 C)(85200 V) 9.10939 1031 kg

= 1.73119 108 m/s . Then the force on it is

An electron is in a uniform magnetic eld B that is directed out of the plane of the page, as shown.

Version 001 Electromagnetism tubman (12126) B e B


Magnetic Field

When the electron is moving in the plane of the page in the direction indicated by the arrow, the force on the electron is directed 1. toward the left 2. toward the top of the page. correct

Which graph best represents the potential dierence E between the ends of the wire as a function of the speed v of the wire? 1. !

2. 3. toward the bottom of the page. 4. into the page.

5. out of the page. 6. toward the right Explanation: The force on the electron is

correct 3.

4. =q = e F vB v B. The direction of the force is thus 5. F = v B , pointing toward the top of the page , using right hand rule for v B , and reversing the direction due to the negative charge on the electron. Wire in Magnetic Field 01 005 10.0 points A wire of constant length is moving in a constant magnetic eld, as shown below. The wire and the velocity vector are perpendicular to each other and also perpendicular to the eld.

Explanation: By Lorentzs Law, = q F v B, the force on the electrons migrating toward one end of the wire increases linearly with the velocity. This indicates that the potential dierence between the ends of the wire will also increase linearly with the velocity. Conceptual 17 Q01

Version 001 Electromagnetism tubman (12126) 006 10.0 points The gure represents two long, straight, parallel wires extending in a direction perpendicular to the page. The current in the left wire runs into the page and the current in the right runs out of the page. a b c

Basic Concept: Magnetic Force on a Current: = IL B F Solution: We have to add up the forces that the magnetic eld produces on each segment of wire. For the segment along the z -axis: z seg = I L B F k ) = I Lz B ( k =0. Therefore, only the segment of wire along the x-axis contributes to the force. This force is xseg = I L B F ) = I Lx B ( k = I Lx B ( ) . Therefore the magnitude of this force is F = (69 A) (6 m) (0.028 T) = 11.592 N.

What is the direction of the magnetic eld created by these wires at location a, b and c? (b is midway between the wires.) 1. up, up, down 2. down, down, up 3. up, zero, down 4. down, zero, up 5. down, up, down 6. up, down, up correct Explanation: By the right-hand rule the left wire has a clockwise eld and the right wire a counterclockwise eld. Force on a Wire Segment 007 10.0 points A segment of wire carries a current of 69 A along the x axis from x = 6 m to x = 0 and then along the z axis from z = 0 to z = 7.9 m. In this region of space, the magnetic eld is equal to 28 mT in the positive z direction. What is the magnitude of the force on this segment of wire?

Parallel Sections of Wires 008 10.0 points Two identical parallel sections of wire are connected parallel to a battery as shown. The two sections of wire are free to move.

When the switch is closed, the wires Correct answer: 11.592 N. Explanation: Let : I B Lx Lz = 69 A , = 28 mT = 0.028 T , = 6 m , and = 7. 9 m . 1. will heat up, and remain motionless. 2. will accelerate away each other. 3. will accelerate towards each other. correct Explanation:

Version 001 Electromagnetism tubman (12126) The currents in both rods move downward, so they are parallel currents that attract, causing them to accelerate toward one another. Force on a Compass Needle 009 10.0 points A compass needle of length , magnetic moment , and mass m is in a constant magnetic eld B . What is the net magnetic force FB on the needle? 1. FB = ( ) B ) 2. FB = ( B 3. FB = ( B ) 4. None of these 5. FB = 0 correct ) 6. FB = ( lB ) 7. FB = ( B 8. FB = B Explanation: Basic Concepts: Loop: =I F Force on a Current ) (d sB

since the sum of displacements around a closed loop is zero. Thus there will be no net magnetic force on a current loop in a uniform magnetic eld. A small arbitrarily shaped current loop is the archetypal magnetic dipole. While a compass needle may not appear much like a current loop, the same principles apply; therefore the compass needle (and all other magnetic dipoles) experiences no net force in a uniform magnetic eld. Electron Gun 010 10.0 points The accelerating voltage that is applied to an electron gun is 85 kV, and the horizontal distance from the gun to a viewing screen is 0.1 m. What is the deection caused by the vertical component of the Earths magnetic eld of strength 4 105 T, assuming that any change in the horizontal component of the beam velocity is negligible. The elemental charge is 1.60218 1019 C and the electrons mass is 9.10939 1031 kg. Correct answer: 0.000203431 m. Explanation: Let : qe = 1.60218 1019 C , d = 0. 1 m , B = 4 105 T , V = 85 kV = 85000 V , and me = 9.10939 1031 kg .

Solution: Consider a small, arbitrarily shaped current loop in a uniform magnetic eld. The net magnetic force on that loop is = I (d ). (This formula given by F sB is obtained by considering the force dF = on a small, straight segment of wire I d sB and adding these forces together for the entire closed loop.) However, since the magnetic eld is constant, we can bring it outside the integral =I F =I 0B = 0, d s B

The velocity v can be obtained from energy conservation K=U 1 me v 2 = qe V 2 v= = 2 qe V me 2 (1.60218 1019 C) (85000 V) 9.10939 1031 kg

= 1.72916 108 m/s .

Version 001 Electromagnetism tubman (12126) From Newtons second law, me a = qe v B vB a = qe me = (1.60218 1019 C) (1.72916 108 m/s) (4 105 T) 9.10939 1031 kg = 1.21651 1015 m/s2 . The time is d 0. 1 m t= = v 1.72916 108 m/s = 5.78316 1010 s , so the deection is 1 1 x = a t2 = (1.21651 1015 m/s2 ) 2 2 (5.78316 1010 s)2 = 0.000203431 m . AP B 1993 FR 3 v1 011 (part 1 of 2) 10.0 points A particle of mass 6.308 1026 kg and charge of 4.8 1019 C is accelerated from rest in the plane of the page through a potential dierence of 461 V between two parallel plates as shown. The particle is injected through a hole in the right-hand plate into a region of space containing a uniform magnetic eld of magnitude 0.274 T. The particle curves in a semicircular path and strikes a detector. q m hole E Let : m = 6.308 1026 kg , V = 461 V , and |q | = 4.8 1019 C . Region of Magnetic Field B +

3. toward the upper right corner of the page 4. toward the upper left corner of the page 5. toward the lower right corner of the page 6. to the right 7. toward the bottom of the page 8. toward the top of the page 9. out of the page correct 10. toward the lower left corner of the page Explanation: +q m + + + hole E B

Because the particle curves down, the direc points down. By the right-hand tion of vB must point out of the page . rule, B 012 (part 2 of 2) 10.0 points What is the magnitude of the force exerted on the charged particle as it enters the region of ? the magnetic eld B Correct answer: 1.10162 1014 N. Explanation:

Which way does the magnetic eld point? 1. into the page 2. to the left

First, we can nd the velocity of the particle by considering the change in kinetic energy of the particle. The change in kinetic energy of

Version 001 Electromagnetism tubman (12126) the charged particle is equal to the work done on it by the potential dierence thus: 1 m v2 = q V 2 2qV m 2 (4.8 C) (461 V) (6.308 1026 kg) 1019

The arrow on a compass is a magnetic north pole. A magnetic north pole will be attracted to a magnetic south pole. This means that the arrow has to be attracted to a magnetic south pole. Therefore, since the compass points to the Earths North Pole, the Earths North Pole is actually a magnetic south pole. MagFieldBetweenTwoWires 014 10.0 points Consider two straight wires each carrying current I , as shown. I

v= =

= 83760.7 m/s .

Let : B = 0.274 T . I Then the force on the particle is given by the Lorentz force law = q F vB = qvB F = (4.8 1019 C) (83760.7 m/s) (0.274 T) = 1.10162 1014 N . EarthAndCompass 013 10.0 points Why does a compass point to the Earths North Pole? 1. The north pole of the compass is attracted to the Earths North Pole due to like-magnetic pole attracting 2. The north pole of the compass is attracted to the Earths magnetic south pole which happens to be near the Earths North Pole correct 3. The arrow of a compass contains an electric charge which is attracted to the electric charge at the Earths North Pole 4. The north pole of the compass is really a south magnetic pole and is attracted to the Earths North Pole Explanation:

What is the direction of the magnetic eld at point R (midpoint between the two wires) caused by the two current carrying wires? 1. There is no magnetic eld correct 2. Out of the page 3. To the left 4. Towards the top wire 5. Into the page 6. To the right 7. Towards the bottom wire Explanation: Use the right-hand rule to determine the direction of the magnetic eld surrounding a long, straight wire carring a current (thumb). We nd that the magnetic eld points Into the page at point R (ngers) due to the bottom wire and points Out of the page for the top wire. The two together are equal and opposite so they add to zero and thus, there is no magnetic eld at point R. AP B 1993 MC 18

Version 001 Electromagnetism tubman (12126) 015 10.0 points Consider a straight wire carrying current I , as shown. R I What is the direction of the magnetic eld at point R caused by the current I in the wire? 1. To the right 2. Into the page correct 3. To the left 4. Away from the wire 5. Out of the page 6. Toward the wire Explanation: Use the right-hand rule to determine the direction of the magnetic eld surrounding a long, straight wire carring a current (thumb). We nd that the magnetic eld points Into the page at point R (ngers). Conceptual 17 Q03 016 10.0 points An electric current runs through a coil of wire as shown. A permanent magnet is located to the right of the coil. The magnet is free to rotate. N i S 2. rotate clockwise 3. rotate counterclockwise correct 4. Unable to determine

Explanation: The magnetic eld inside the coil points to the left, so the right side of the coil is a S pole. This will attract the N pole of the bar magnet, causing it to rotate counterclockwise. Electromagnetism 11 017 10.0 points A long coil of wire with many loops is called a 1. motor. 2. galvanometer. 3. solenoid. correct 4. transformer. 5. generator. Explanation: AP B 1993 MC 41 018 10.0 points A copper wire of constant length is moving in a constant magnetic eld, as shown.

r B
v

Pivot Figure: The wire and the velocity vector v are perpendicular (in the horizontal plane) to each other and are both perpendicular to the mag (vertical). netic eld B Which of the following graphs best represents the magnitude of the emf E between the ends of the wire as a function of the speed v of the wire?

What will happen to the magnet if its original orientation is as shown in the gure, with the current coming in on the front side of the solenoid, and then looping around the back? 1. remain still

Version 001 Electromagnetism tubman (12126) 5 4 3 2 1 0 0 1 2 3 4 5 6 7 8 910 Velocity (m/s) 5 4 3 2 1 0 0 1 2 3 4 5 6 7 8 910 Velocity (m/s) 5 4 3 2 1 0 0 1 2 3 4 5 6 7 8 910 Velocity (m/s) 5 4 3 2 1 0 0 1 2 3 4 5 6 7 8 910 Velocity (m/s) 5 4 3 2 1 0 0 1 2 3 4 5 6 7 8 910 Velocity (m/s) 5 4 3 2 1 0 0 1 2 3 4 5 6 7 8 910 Velocity (m/s) E (V) E (V) E (V) E (V) E (V) 5 4 3 2 1 0 0 1 2 3 4 5 6 7 8 910 Velocity (m/s) correct

E (V)

1.

7.

2.

Explanation: As the conductor moves with velocity v , the charge carriers experience a magnetic force Fmag . This force leads to a separation of charge carriers of opposite sign. This in turn creates an electric eld EH that leads to an electric force FE opposing the magnetic force. In equilibrium these two forces balance out. Hence FE = Fmag q EH = q v B ; with EH = E one gets E = EH = v B , E v, or

3.

4.

5.

since B and are constants. Alternative Solution: From Faradays law, E = B v = E v , since B and are constants. Thus the correct graph is 5 4 3 2 1 0 0 1 2 3 4 5 6 7 8 910 Velocity (m/s) Bar in a Field 01 019 10.0 points Consider the arrangement shown. Assume that R = 8.69 , = 0.784 m, and a uniform 1.43 T magnetic eld is directed into the page. E (V)

6.

E (V)

Fapp

Version 001 Electromagnetism tubman (12126) At what speed should the bar be moved to produce a current of 0.729 A in the resistor? Correct answer: 5.65061 m/s. Explanation: Let : R = 8.69 , = 0.784 m , B = 1.43 T , and I = 0.729 A . Conceptual 18 Q15 021 10.0 points Consider a transformer. What is true?

1. A transformer does not work with direct current. correct 2. A transformer works with direct current. Explanation: A transformer does not work with direct current because electromagnetic induction requires a changing magnetic eld. The magnetic eld in a transformer with direct current would be constant.

The emf inside the loop can be calculated using Ohms Law E =IR and the motional emf is E = Bv, so the velocity of the bar is v=

E IR (0.729 A)(8.69 ) = = = 5.65061 m/s . B B (1.43 T)(0.784 m)

Holt SF 22Rev 36 020 10.0 points A transformer is used to convert 120 V to 12 V in order to power a toy electric train. There are 240 turns in the primary. How many turns should there be in the secondary? Correct answer: 24 turns. Explanation: Let : V1 = 120 V , V2 = 12 V , and N1 = 240 turns .

V1 V2 = N1 N2 V2 N1 N2 = V1 (12 V) (240 turns) = 120 V = 24 turns .

You might also like